question stems Flashcards

1
Q

Which one of the following principles is best illustrated by the study described above

A

Principles

How well did you know this?
1
Not at all
2
3
4
5
Perfectly
2
Q

Which one of the following, if true, most strengthens the argument?

A

Strengthen assumption

How well did you know this?
1
Not at all
2
3
4
5
Perfectly
3
Q

The Conclusion is must strongly supported by the reasoning in the argument if which one of the following is assumed?

A

Assumption

How well did you know this?
1
Not at all
2
3
4
5
Perfectly
4
Q

Which one of the following, if true, provides the most support for the industry analysts’ prediction?

A

Strengthen Assumption

How well did you know this?
1
Not at all
2
3
4
5
Perfectly
5
Q

The area resident’s argument is flawed in that it

A

flaw

How well did you know this?
1
Not at all
2
3
4
5
Perfectly
6
Q

If all of the statements above are true, which one of the following must be true?

A

Inference

How well did you know this?
1
Not at all
2
3
4
5
Perfectly
7
Q

The Flawed pattern of reasoning in which one of the following is most closely parallel to that in MacNeil’s argument?

A

Parallel

How well did you know this?
1
Not at all
2
3
4
5
Perfectly
8
Q

The reasoning in the argument is flawed in that the argument

A

Flaw

How well did you know this?
1
Not at all
2
3
4
5
Perfectly
9
Q

Which of one of the following, If assumed, enables the argument’s conclusion to be properly drawn?

A

Assumption

How well did you know this?
1
Not at all
2
3
4
5
Perfectly
10
Q

Which one of the following most accurately states the conclusion drawn in the argument

A

main point

How well did you know this?
1
Not at all
2
3
4
5
Perfectly
11
Q

which one of the following most accurately describes the reporter’s error in reasoning?

A

Flaw

How well did you know this?
1
Not at all
2
3
4
5
Perfectly
12
Q

Which one of the following is an assumption on which the paleontologist’s argument depends?

A

Assumption (Necessary)

How well did you know this?
1
Not at all
2
3
4
5
Perfectly
13
Q

Which one of the following most closely conforms to the principle illustrated above

A

Principle (Identify and Apply)

How well did you know this?
1
Not at all
2
3
4
5
Perfectly
14
Q

The Music Critic’s Response To the Music professor’s argument

A

Method Of Argument

How well did you know this?
1
Not at all
2
3
4
5
Perfectly
15
Q

The speaker’s main conclusion logically follows if Which one of the following is assumed?

A

Assumption (sufficient?)

How well did you know this?
1
Not at all
2
3
4
5
Perfectly
16
Q

Each of the following assignments of computer programmers is consistent both with principle expressed by Vanessa and with the principle expressed by Jo Except:

A

Principle (identify and apply)

17
Q

Which one of the following, if true, most helps explain why the astronomer’s estimates of the stars distance from earth help resolve the earlier conflict between the ages of these stars and the age of the universe?

A

Paradox

18
Q

Which one of the following is most strongly supported by the information above?

A

Inference

19
Q

Which one of the following, if true, most seriously weakens the manager’s argument?

A

Weakener (Assumption)

20
Q

Which one of the following if , must seriously undermines the editor’s statement?

A

Weakener (assumption)

21
Q

which one of the Following principles, if true, most helps to justify the consumers advocate’s argumentation?

A

Principle (Identify)

22
Q

The statement above, if true, provide a basis for rejecting which one of the following claims?

A

Inference?

23
Q

The university president’s argument requires the assumption that?

A

Assumption (basic)

24
Q

Which one of the following principles, if valid, would most help to justify the reasoning in the editorial?

A

Principle (identify)

25
Q

Which one of the following is best supported by the information above?

A

Inference MBT

26
Q

If the shareholder’s statements are true, which one of the following is most strongly supported by them?

A

Inference MBT

27
Q

Which one of the following can be properly inferred from the ecologist’s statements?

A

Inference MBT

28
Q

If the statements above are true, then each of the following could also be true EXCEPT

A

Inference (mbf/impossible- wrong answer).

29
Q

If the statements above are true, which one of the following most also be true on the basis of them?

A

Inference